Finite Math and Applied Calculus (6th Edition)

Published by Brooks Cole
ISBN 10: 1133607705
ISBN 13: 978-1-13360-770-0

Chapter 3 - Section 3.2 - Using Matrices to Solve Systems of Equations - Exercises - Page 204: 16

Answer

$(6,-1,-1)$

Work Step by Step

Write the augmented matrix and, using row transformations arrive at the reduced row echelon form. ----------- $\left[\begin{array}{llll} 1 & 2 & 0 & 4\\ 0 & 1 & -1 & 0\\ 1 & 3 & -2 & 5 \end{array}\right]\begin{array}{l} .\\ .\\ -R_{1}. \end{array}$ ... clear column 1, $\left[\begin{array}{llll} 1 & 2 & 0 & 4\\ 0 & 1 & -1 & 0\\ 0 & 1 & -2 & 1 \end{array}\right]\begin{array}{l} -2R_{2}.\\ .\\ -R_{2}. \end{array}$ ... clear column 2, $\left[\begin{array}{llll} 1 & 0 & 2 & 4\\ 0 & 1 & -1 & 0\\ 0 & 0 & -1 & 1 \end{array}\right]\begin{array}{l} +2R_{3}.\\ -R_{3}.\\ \div(-1). \end{array}$ ... leading 1 in row 3, ... clear column 3, $\left[\begin{array}{llll} 1 & 0 & 0 & 6\\ 0 & 1 & 0 & -1\\ 0 & 0 & 1 & -1\\ & & & \end{array}\right]$ Solution: $(6,-1,-1)$
Update this answer!

You can help us out by revising, improving and updating this answer.

Update this answer

After you claim an answer you’ll have 24 hours to send in a draft. An editor will review the submission and either publish your submission or provide feedback.